LSAT and Law School Admissions Forum

Get expert LSAT preparation and law school admissions advice from PowerScore Test Preparation.

 Administrator
PowerScore Staff
  • PowerScore Staff
  • Posts: 8917
  • Joined: Feb 02, 2011
|
#81375
Complete Question Explanation

The correct answer choice is (B).

Answer choice (A):

Answer choice (B): This is the correct answer choice.

Answer choice (C):

Answer choice (D):

Answer choice (E):

This explanation is still in progress. Please post any questions below!
 cmorris32
  • Posts: 92
  • Joined: May 05, 2020
|
#76085
Hi PowerScore!

I am having a difficult time finding textual support for answer choice B, which is the correct answer, stating that a future study would be informative about effects of social constraints on medieval women's medical education.

I eliminated C, D, and E, and eventually chose answer choice A, since nothing in the passage relates to information about medical education. I thought answer choice A was supported by the quote "Information about economic rivalry... role in medieval society," which is the last sentence of the passage.

Can you please explain why answer choice B is correct, and why my identification of the textual support for answer choice A is incorrect?

Thank you!
Caroline :-D
 Luke Haqq
PowerScore Staff
  • PowerScore Staff
  • Posts: 742
  • Joined: Apr 26, 2012
|
#76258
Hi Caroline!

Between answer choices (A) and (B), the first of these seems to general, whereas the second is more in line with the passage's focus on women practicing medicine in the middle ages.

With the question stem asking about how future studies might be "more informative about medieval women medical practitioners," answer choice (A) refers to a future study of "the effect of social change on the political and economic structure of medieval society." The passage emphasizes that too little research has been done on medieval women medical practitioners, so one would expect a future study that is endorsed by the author of this passage to be one that gives close attention to that particular group. This answer choice fails to do that, because of how generally it is stated. The text around lines 63-65 about economic rivalry certainly may be relevant, but it's not enough to make answer choice (A) correct--that answer choice would have at least had to have mentioned some rivalry between women and men, but it didn't do so.

Answer (B), by contrast, does contain the right level of specificity--it refers directly to issues pertaining to women and their ability to practice medicine. A future study on "the effect of social constraints on medieval women's access to a medical education" would address both of these matters. Other than generally in terms of the function and purpose of the passage, for more direct textual support, I'd look around lines 60-63: "Future studies might also make profitable use of analyses developed in other areas of women's history as a basis for exploring the social context of women's medical practice." This seems to provide direct support for answer choice (B), since it applies thinking about broader social constraints to an examination specifically of medieval women's medical practice.

Get the most out of your LSAT Prep Plus subscription.

Analyze and track your performance with our Testing and Analytics Package.